The Student Room Group

Causal functions/Laplace transformations help

Hi can anyone help with this question:

Find the area under the curve 1/(t+3) [u(t-1)-u(t-3)] between t=-2 and t=2.5.

I just have no idea what to do. I've made some attempts but can't get it. So if someone knows how to do this and can show me a worked answer that would be much appreciated.

The answer is 0.3185 if you want to check what you've done is right.

Thanks!
Reply 1
Original post by Should_be_studying
Hi can anyone help with this question:

Find the area under the curve 1/(t+3) [u(t-1)-u(t-3)] between t=-2 and t=2.5.

I just have no idea what to do. I've made some attempts but can't get it. So if someone knows how to do this and can show me a worked answer that would be much appreciated.

The answer is 0.3185 if you want to check what you've done is right.

Thanks!


You'll need to tell us what the function u(t) does first!

(It may be standard on your course, but most people won't know what it is!!)
Original post by davros
You'll need to tell us what the function u(t) does first!

(It may be standard on your course, but most people won't know what it is!!)


Yeah sorry, it's standard!

u(t)= 1 if t>= 0
= 0 if t < 0
Original post by davros
You'll need to tell us what the function u(t) does first!

(It may be standard on your course, but most people won't know what it is!!)


In the context of a Laplace transform, it is almost certainly the Heaviside step function, though the question doesn't seem to require knowledge of Laplace transforms per se.

The difference of two step functions gives a square pulse, so the it's trivial to work out the area in this case.
Reply 4
Original post by atsruser
In the context of a Laplace transform, it is almost certainly the Heaviside step function, though the question doesn't seem to require knowledge of Laplace transforms per se.

The difference of two step functions gives a square pulse, so the it's trivial to work out the area in this case.


I suspected as much, so it's not really a Laplace transform problem at all - the OP may have attracted more help if it had just been presented as an integration problem :smile:

Quick Reply

Latest